0

Normal to a hyperabola- class-XII

Description: normal to a hyperabola
Number of Questions: 35
Created by:
Tags: hyperbola two dimensional analytical geometry-ii maths
Attempted 0/35 Correct 0 Score 0

The equation of the curve which is such that the protion of the axis of x cut off between the origin and tangent at any point is proportional to the ordinate of that point is _______________.

  1. $\log x = b y ^ { 2 } + a$

  2. $x = y ( a + b \log y )$

  3. $x = y ( b - a \log y )$

  4. None of these


Correct Option: C

The hyperbola $\frac{{{x^2}}}{{{a^2}}} - \frac{{{y^2}}}{{{b^2}}} = 1$, normals are drawn to curve $\left( {{{\left( {\frac{{{x^2}}}{{{a^2}}} - \frac{{{y^2}}}{{{b^2}}}} \right)}^2} - 1} \right)\left( {\frac{{{x^2}}}{{{a^2}}} - \frac{{{y^2}}}{{{b^2}}}} \right) = 0$.
Find the sum;  of abscissa of foot of all such normals.

  1. $\frac{{6{a^2}h}}{{\left( {{a^2} + {b^2}} \right)}}$

  2. $\frac{{8{a^2}h}}{{\left( {{a^2} + {b^2}} \right)}}$

  3. $\frac{{6a{h^2}}}{{\left( {{a^2} + {b^2}} \right)}}$

  4. $\frac{{8a{h^2}}}{{\left( {{a^2} + {b^2}} \right)}}$


Correct Option: B

If the straight line $(a - 2) x - by + 4 = 0$ is normal to the hyperbola $xy = 1$ then which of the followings does not hold?

  1. $a > 1, b > 0$

  2. $a > 1, b < 0$

  3. $a < 1, b < 0$

  4. $a < 1, b > 0$


Correct Option: A,C
Explanation:

Every normal to $xy = 1$ must have positive slope as $\dfrac {-dx}{dy} = x^{2}$. So $\dfrac {a - 1}{b} > 0$.

The normal to the hyperbola $4x^2-9y^2=36$ meets the axes in $M$ and $N$ and the lines $MP$, $NP$ are drawn right angles at the axes. The locus of $P$ is the hyperbola 

  1. $9x^2-4y^2=169$

  2. $4x^2-9y^2=169$

  3. $3x^2-4y^2=169$

  4. $None\ of\ these$


Correct Option: D
Explanation:

$\dfrac {x^2}9-\dfrac {y^2}4=1$.Let $P(x _1, y _1)$ be the point on hyperbola.

Eqn of the normal is$\dfrac {a^2x}{x _1}-\dfrac {b^2y}{y _1}=a^2b^2\M=\left( \dfrac { { a }^{ 2 }-{ b }^{ 2 } }{ { a }^{ 2 } }  \right) { x } _{ 1 }=x\N=\left( \dfrac { { a }^{ 2 }-{ b }^{ 2 } }{ { a }^{ 2 } }  \right) { y } _{ 1 }=y\P=(x, y)$$x _1=\dfrac {a^2(x)}{a^2-b^2}$ $(x _1, y _1)$ lies at hyperbola.$y _1=\dfrac {b^2(y)}{a^2-b^2}$ $(x _1, y _1)$ lies at hyperbola.Now, $a^2=9, b^2=4$Therefore, $x _1=\dfrac {9x}5, y _1=\dfrac {4y}5$$\dfrac {x _1^2}9-\dfrac {y _1^2}4=1\\left(\dfrac {9x}5\right)^2\dfrac {x _1^2}9-\left(\dfrac {4y}5\right)^2\dfrac {y _1^2}4=1\\implies 9x^2-4y^2=25$

A normal to the hyperbola, $4x^2-9y^2=36$ meets the co-ordinate axes x and y at A and B, respectively. If the parallelogram $OABP$($O$ being the origin) is formed, then the locus of $P$ is?

  1. $4x^{2}+9y^{2}=121$

  2. $9x^{2}+4y^{2}=169$

  3. $4x^{2}-9y^{2}=121$

  4. $9x^{2}-4y^{2}=169$


Correct Option: D

Equation of the normal to the hyperbola $3x^2-y^2=3$ at $(2, -3)$ is?

  1. $x-2y-8=0$

  2. $3x-2y-12=0$

  3. $x+2y+4=0$

  4. $3x+2y-14=0$


Correct Option: A

Line x cos$\alpha $+yin$\alpha $=p is a normal to the hyperbola $\frac{x^{2}}{a^{2}}-\frac{y^{2}}{b^{2}}=1 $, if

  1. $a^{2}sec^{2}\alpha -b^{2}cosec^{2}\alpha =\frac{(a^{2}+b^{2})^{2}}{p^{^{2}}}$

  2. $a^{2}sec^{2}\alpha+b^{2}cosec^{2}\alpha =\frac{(a^{2}+b^{2})^{2}}{p^{^{2}}}$

  3. $a^{2}cos^{2}\alpha -b^{2}sin^{2}\alpha =\frac{(a^{2}+b^{2})^{2}}{p^{^{2}}}$

  4. $a^{2}cos^{2}\alpha+b^{2}sin^{2}\alpha =\frac{(a^{2}+b^{2})^{2}}{p^{^{2}}}$


Correct Option: A

Line $ x \cos \alpha + y \sin \alpha = p $ is a normal to the hyperbola $ \frac { x ^ { 2 } } { a ^ { 2 } } - \frac { y ^ { 2 } } { b ^ { 2 } } = 1 $, if 

  1. $ a ^ { 2 } \sec ^ { 2 } \alpha - b ^ { 2 } \csc ^ { 2 } \alpha = \frac { \left( a ^ { 2 } + b ^ { 2 } \right) ^ { 2 } } { p ^ { 2 } } $

  2. $ a ^ { 2 } \sec ^ { 2 } x + b ^ { 2 } \csc ^ { 2 } \alpha = \frac { \left( a ^ { 2 } + b ^ { 2 } \right) ^ { 2 } } { p ^ { 2 } } $

  3. $ a ^ { 2 } \cos ^ { 2 } \alpha - b ^ { 2 } \sin ^ { 2 } \alpha = \frac { \left( a ^ { 2 } + b ^ { 2 } \right) ^ { 2 } } { p ^ { 2 } } $

  4. $ a ^ { 2 } \cos ^ { 2 } \alpha + b ^ { 2 } \sin ^ { 2 } \alpha = \frac { \left( a ^ { 2 } + b ^ { 2 } \right) ^ { 2 } } { p ^ { 2 } } $


Correct Option: A

A straight line is drawn parallel to the conjugate axis of the hyperbola $\dfrac{{{x^2}}}{{{a^2}}} - \dfrac{{{y^2}}}{{{b^2}}} = 1$ to meet it and the conjugate hyperbola respectively in the point $P$ and $Q$. The normals at $p$ and $Q$ to the curves meet on 

  1. $x-axis$

  2. $y-axis$

  3. $y=x$

  4. $y=-x$


Correct Option: A
Explanation:
Equation of hyperbola $\to \dfrac {x^2}{9^2}-\dfrac {y^2}{6^2}=1$
Conjugate hyperbola $\to \dfrac {y^2}{6^2}-\dfrac {x^2}{9^2}=1$
Line $||$ to conjugate axis of hyperbola $(y-axis)$ is drawn to meet conjugate axis at $P$ & $Q$ which are symmetric points about $x-$axis then
$P(a\tan \theta , b\sin theta)$
$Q(a \tan \theta , -b\sin \theta)$
Normal at $P\to \dfrac {ax}{\tan \theta} +\dfrac {6y}{\sec \theta}=a^2+b^2$
Normal at $Q\to \dfrac {ax}{\tan \theta}-\dfrac {6y}{\sec \theta}=a^2+b^2$
Let us find out interrection
$\dfrac {b}{\sec \theta}=\dfrac {-6y}{\sec \theta} $
$y=0$
$9+$ lies on $x-$axis
$A$ is correct


If the normal at $\left (ct _1,\dfrac { c}{t _1}\right)$ on the hyperbola $xy = c^2$ cuts the hyperbola again at $\left (ct _2, \dfrac {c}{t _2}\right)$, then $t _2^3 t _2$ $=$ 

  1. $2$

  2. $-2$

  3. $-1$

  4. $1$


Correct Option: C
Explanation:

The equation of hyperbola is $xy=c^2$ and point $(ct _1,\dfrac{c}{t _1})$ lies on it.


Let us find the equation of the normal.


Equation of hyprbola can be written as $y=\dfrac{c^2}{x}$ and therefore slope of tangent is given by first derivative i.e. $\dfrac{dy}{dx}=−\dfrac{c^2}{x^2}$


hence slope of normal is given by $\dfrac{x^2}{c^2}$ and at $(ct _1,\dfrac{c}{t _1})$ is $t^2$ and its equation is


$y=t _1^2(x−ct _1)+\dfrac{c}{t _1}$


or $xt _1^3−yt _1−ct _1^4+c=0$


As this passes through $(ct _2,\dfrac{c}{t _2})$


$ct _2t _1^3−\dfrac{c}{t _2}t _1−ct _1^4+c=0$


or $ct _1^3(t _2−t _1)+\dfrac{c}{t _2}(t _2−t _1)=0$


as $t _1\neq t _2, t _1−t _2\neq 0$ and dividing by it we get


$ct _2^3=−\dfrac{c}{t _2}$


Or $ t _2^3t _2=−1$

If the tangent and normal to a rectangular hyperbola cut off intercepts $x _1$ and $x _2$ on one axis and $y _1$ and $y _2$ on the other axis, then

  1. $x _1y _1+x _2y _2=0$

  2. $x _1y _2+x _2y _1=0$

  3. $x _1x _2+y _1y _2=0$

  4. none of these


Correct Option: C
Explanation:

Assume rectangular hyperbola is $xy = c^2$
Thus equation of tangent and normal at any point 't' are,
$\cfrac{x}{t}+ty=2c$ and $ y-\cfrac{c}{t}=t^2(x-ct)$
Now putting $y=0$ in both the equation we get, $x _1=2ct, x _2=ct-\cfrac{c}{t^3}$
and putting $x=0$ we get, $y _1=\cfrac{2c}{t}, y _2=\cfrac{c}{t}-ct^3$
$\Rightarrow x _1x _2+y _1y _2=2ct(ct-\cfrac{c}{t^3})+\cfrac{2c}{t}(\cfrac{c}{t}-ct^3)=0$
Hence, option 'C' is correct.

The number of normal to the hyperbola $\cfrac { { x }^{ 2 } }{ { a }^{ 2 } } -\cfrac { { y }^{ 2 } }{ { b }^{ 2 } } =1$ from an external point is

  1. $2$

  2. $4$

  3. $6$

  4. $5$


Correct Option: B
Explanation:

Given hyperbola is, $\displaystyle \cfrac{x^{2}}{a^2} - \cfrac{y^{2}}{b^2} = 1$
The general equation of normal to hyperbola with slope 'm' is given by,
$y = mx\pm\cfrac{(a^2+b^2)m}{\sqrt{a^2-b^2m^2}}$
Let any external point through wich this line is passing is $P(x _1,y _1)$
$\Rightarrow (y _1-mx _1)^2=\cfrac{(a^2+b^2)^2m^2}{a^2-b^2m^2}$
$\Rightarrow (y _1-mx _1)^2(a^2-b^2m^2)=(a^2+b^2)^2m^2$
Clearly, this is a polynomial of degree four so maximum number of normal that can be drawn from point P (any external point) to the hyperbola is 4 corresponding to four roots of m.
Hence, option 'B' is correct.

The normal to the rectangular hyperbola $xy=-c^2$ at the point $'t _1'$ meets the curve again at the point $'t _2'$. The value of $t _1^3 \cdot t _2$ is

  1. $1$

  2. $c$

  3. $-c$

  4. $-1$


Correct Option: D
Explanation:

The equation of the normal $t _{1}$is  $y-\dfrac{c}{t _{1}}=t _{1}^{2}(x-ct _{1})$

If this passes through $\left ( ct _{2},\dfrac{c}{t _{2}} \right )$

$\dfrac{\mathrm{c}}{\mathrm{t} _{2}}-\dfrac{\mathrm{c}}{\mathrm{t} _{\mathrm{t}}}=\mathrm{t} _{1}^{2}(\mathrm{c}\mathrm{t} _{2}-\mathrm{c}\mathrm{t} _{1})$

$\displaystyle



\Rightarrow-\dfrac{1}{\mathrm{t} _{\mathrm{t}}\mathrm{t} _{2}}=\mathrm{t} _{1}^{2}\Rightarrow

1+\mathrm{t} _{\mathrm{t}}^{3}\mathrm{t} _{2}=0$

$\Rightarrow t _1^3t _2=-1$

Hence, option 'D' is correct.

If the normal at '$\theta $' on the hyperbola $\cfrac { { x }^{ 2 } }{ { a }^{ 2 } } -\cfrac { { y }^{ 2 } }{ { b }^{ 2 } } =1$ meets the transverse axis at $G$  and $A$ and $A'$ are the vertices of the hyperbola, then $AG.A'G$ $=$

  1. ${ a }^{ 2 }\left( { e }^{ 2 }\sec ^{ 2 }{ \theta } -1 \right) $

  2. ${ a }^{ 2 }\left( { e }^{ 4 }\sec ^{ 2 }{ \theta } -1 \right) $

  3. ${ a }^{ 2 }\left( { e }^{ 4 }\sec ^{ 2 }{ \theta } +1 \right) $

  4. none of these


Correct Option: B
Explanation:

The equation of the normal at $\left( a\sec { \theta  } ,b\tan { \theta  }  \right) $ to the given hyperbola is
$ax\cos { \theta  } +by\cot { \theta  } =\left( { a }^{ 2 }+{ b }^{ 2 } \right) $
This meets the transverse axis (i.e) at $G$. So, the coordinates of $G$ are $\left{ \cfrac { { a }^{ 2 }+{ b }^{ 2 } }{ a } \sec { \theta  }
,0 \right} $
The coordinates of the vertices $A$ and $A'$ are $A(a,0)$ and $A'(-a,0)$ respectively
$\therefore

\quad AG.A'G=\left( -a+\cfrac { { a }^{ 2 }+{ b }^{ 2 } }{ a } \sec {

\theta  }  \right) \left( a+\cfrac { { a }^{ 2 }+{ b }^{ 2 } }{ a } \sec

{ \theta  }  \right)$
 $\Rightarrow AG.A'G=\left( -a+a{ e }^{ 2 }\sec { \theta  }  \right) \left( a+a{ e }^{ 2 }\sec { \theta  }  \right) $
$={ a }^{ 2 }\left( { e }^{ 4 }\sec ^{ 2 }{ \theta  } -1 \right) $
Hence, option 'B' is correct.

If the normal at $'\theta'$ on the hyperbola $\displaystyle \frac{x^2}{a^2} - \frac{y^2}{b^2} = 1$ meets the transverse axis at G, and A and A' are the vertices of the hyperbola, then AG.A'G $=$

  1. $a^2 (e^2 sec^2 \theta -1)$

  2. $a^2 (e^4 sec^2 \theta - 1)$

  3. $a^2 (e^4 sec^2 \theta + 1)$

  4. none of these


Correct Option: B
Explanation:

The equation of the normal at $\left( a\sec { \theta  } ,b\tan { \theta  }  \right) $ to the given hyperbola is
$ax\cos { \theta  } +by\cot { \theta  } =\left( { a }^{ 2 }+{ b }^{ 2 } \right) $
This meets the transverse axis (i.e) at $G$. So, the coordinates of $G$ are $\left{ \cfrac { { a }^{ 2 }+{ b }^{ 2 } }{ a } \sec { \theta  }

,0 \right} $
The coordinates of the vertices $A$ and $A'$ are $A(a,0)$ and $A'(-a,0)$ respectively
$\therefore

\quad AG.A'G=\left( -a+\cfrac { { a }^{ 2 }+{ b }^{ 2 } }{ a } \sec {

\theta  }  \right) \left( a+\cfrac { { a }^{ 2 }+{ b }^{ 2 } }{ a } \sec

{ \theta  }  \right)$
 $\Rightarrow AG.A'G=\left( -a+a{ e }^{ 2 }\sec { \theta  }  \right) \left( a+a{ e }^{ 2 }\sec { \theta  }  \right) $
$={ a }^{ 2 }\left( { e }^{ 4 }\sec ^{ 2 }{ \theta  } -1 \right) $
Hence, option 'B' is correct.

The equation of normal at $\left( at,\dfrac { a }{ t }  \right)$ to the hyperbola $xy={ a }^{ 2 }$ is ________________________.

  1. ${ xt }^{ 3 }-yt+{ at }^{ 4 }-a=0$

  2. ${ xt }^{ 3 }-yt-{ at }^{ 4 }+a=0$

  3. ${ xt }^{ 3 }+yt+{ at }^{ 4 }-a=0$

  4. ${ xt }^{ 3 }+yt-{ at }^{ 4 }-a=0$


Correct Option: B

The normal at P to a hyperbola of eccentricity e, intersects its transverse and conjugate axes at L and M respectively. If locus of the mid-point of LM is a hyperbola, then eccentricity of the hyperbola is

  1. $\displaystyle \frac{e + 1}{e-1}$

  2. $\displaystyle \frac{e}{\sqrt{e^2 - 1}}$

  3. $e$

  4. none of these


Correct Option: B
Explanation:

The equation of the normal at $P\left( a\sec { \theta  } ,b\tan {

\theta  }  \right) $ to the hyperbola $\cfrac { { x }^{ 2 } }{ { a

}^{2 } } -\cfrac { { y }^{ 2 } }{ { b }^{ 2 } } =1$ is
$ax\cos { \theta  } +by\cot { \theta  } ={ a }^{ 2 }+{ b }^{ 2 }$
This intersects the transverse and conjugate axes at $ L\left( \cfrac { {

a}^{ 2 }+{ b }^{ 2 } }{ a } \sec { \theta  } ,0 \right) $ and

$M\left(0,\cfrac { { a }^{ 2 }+{ b }^{ 2 } }{ { b }^{  } } \tan {

\theta  }  \right) $ respectively
Let $N(h,k)$. then $h=\cfrac { { a }^{ 2 }+{ b }^{ 2 } }{ b } \sec { \theta  } $ and
$k=\cfrac { { a }^{ 2 }+{ b }^{ 2 } }{ { b }^{  } } \tan { \theta  } $
$\Rightarrow

\sec { \theta  } =\cfrac { 2ah }{ { a }^{ 2 }+{ b }^{ 2 } } \quad

,\quad \tan { \theta  } =\cfrac { 2bk }{ { a }^{ 2 }+{ b }^{ 2 } }

\quad$
$\therefore \sec ^{ 2 }{ \theta  } -\tan ^{ 2 }{ \theta  }

=1\quad \Rightarrow 4{ a }^{ 2 }{ h }^{ 2 }-4{ b }^{ 2 }{ k }^{ 2 }={

\left( { a }^{ 2 }+{ b }^{ 2 } \right)  }^{ 2 }$
Thus the locus of

$(h,k)$ is $\Rightarrow 4{ a }^{ 2 }{ x }^{ 2 }-4{ b }^{ 2 }{ y }^{

2}={ \left( { a }^{ 2 }+{ b }^{ 2 } \right)  }^{ 2 }\quad $
Let ${ e } _{ 1 }$ be the eccentricity of this hyperbola. Then
${{

e } _{ 1 } }^{ 2 }=1+\cfrac { { a }^{ 2 } }{ { b }^{ 2 } } =\cfrac { {a

}^{ 2 }+{ b }^{ 2 } }{ { b }^{ 2 } } =\cfrac { { a }^{ 2 }{ e }^{ 2 }}{ {

a }^{ 2 }({ e }^{ 2 }-1) } $
$\Rightarrow { e } _{ 1 }=\displaystyle \frac { e }{ \sqrt { { e }^{ 2 }-1 }  } $

The maximum number of normals to the hyperbola $\displaystyle \frac{x^2}{a^2} - \frac{y^2}{b^2}=1$ from an external point is :

  1. $2$

  2. $4$

  3. $6$

  4. $5$


Correct Option: B
Explanation:

Given hyperbola is, $\displaystyle \cfrac{x^{2}}{a^2} - \cfrac{y^{2}}{b^2} = 1$
The general equation of normal to hyperbola with slope $m$ is given by,
$y = mx\pm\cfrac{(a^2+b^2)m}{\sqrt{a^2-b^2m^2}}$
Let any external point through wich this line is passing is $P(x _1,y _1)$
$\Rightarrow (y _1-mx _1)^2=\cfrac{(a^2+b^2)^2m^2}{a^2-b^2m^2}$
$\Rightarrow (y _1-mx _1)^2(a^2-b^2m^2)=(a^2+b^2)^2m^2$
Clearly, this is a polynomial of degree four so maximum number of normal that can be drawn from point $P$ (any external point) to the hyperbola is $4$ corresponding to four roots of $m$.
Hence, option 'B' is correct.

Set of value of h for which the number of distinct common normals of $(x-2)^{ 2 }=4 (y-3)$ and ${ x }^{ 2 }+{ y }^{ 2 }-2x-hy-c=0$ where, $\left( c>0 \right) $ is 3, is 

  1. $\left( 2,\infty \right) $

  2. $\left( 4,\infty \right) $

  3. $(2, 4)$

  4. $\left( 10,\infty \right) $


Correct Option: A
Explanation:
Given,

$(x-2)^2=4(y-3)$

comparing the given equation with general formula, we get,

$(x-h)^2=4(y-k)$

$\Rightarrow h=2$

$x^2+y^2-2x-hy-3=0$

here the h can take value upto infinity,

Therefore the set of values of $h=(2,\infty )$

The length of sub normal to the curve $xy={ a }^{ 2 }$ at (x,y) on it varies at

  1. ${ x }^{ 2 }$

  2. ${ y }^{ 2 }$

  3. ${ x }^{ 3 }$

  4. ${ y }^{ 3 }$


Correct Option: D
Explanation:
Given,

$xy=a^2$

$\Rightarrow x\dfrac{dy}{dx}+y=0$

$\therefore \dfrac{dy}{dx}=-\dfrac{y}{x}$

Now,

Sub normal $=y\dfrac{dy}{dx}$

$=y\left ( -\dfrac{y}{x} \right )$

$=-\dfrac{y^2}{x}$

$=-\dfrac{y^2}{\frac{a^2}{y}}$

$=-\dfrac{y^3}{a^2}$

$\therefore SN\propto y^3$

Let $P (a\sec \theta , b\tan \theta ) $ and $Q\left ( a\sec \phi , b\tan \phi  \right )$ where $\theta +\phi =\pi /2$, be two points on the hyperbola $x^{2}/a _{2}-y _{2}/b _{2}=1$. If (h, k) is the point of intersection of normals at P and Q, then k is equal to

  1. $\displaystyle \frac{a^{2}+b^{2}}{a}$

  2. $\displaystyle -\left [ \frac{a^{2}+b^{2}}{a} \right ]$

  3. $\displaystyle \frac{a^{2}+b^{2}}{b}$

  4. $\displaystyle -\left [ \frac{a^{2}+b^{2}}{b} \right ]$


Correct Option: D
Explanation:

Equations of the normal at P is $ax+bycosec\theta =\left ( a^{2}+b^{2} \right )\sec \theta $          (i)
and the equation of the normal at $Q\left ( a\sec \phi , b\sec \phi  \right )$ is
$ax+by cosec\phi =\left ( a^{2}+b^{2} \right )\sec \phi $          (ii)
Subtracting (ii) from (i) we get

   $\displaystyle y=\frac{a^{2}+b^{2}}{b}.\frac{\sec \theta -\sec \phi }{cosec \theta -cosec \phi }$

So $\displaystyle k=y=\frac{a^{2}+b^{2}}{b}.\frac{\sec \theta -\sec

\left ( \pi /2-\theta  \right )}{cosec \theta -cosec \left ( \pi

/2-\theta  \right )}$          $\left [ \because \theta +\phi =\pi /2

\right ]$

     $\displaystyle =\frac{a^{2}+b^{2}}{b}.\frac{\sec

\theta -cosec \theta }{cosec \theta -\sec \theta }=-\left [

\frac{a^{2}+b^{2}}{b} \right ]$

Find the equation of normal to the hyperbola $\displaystyle \frac{x^2}{25}\, -\, \displaystyle \frac{y^2}{16}\, =\, 1$ at $(5, 0)$.

  1. $y = 0$

  2. $y=-1$

  3. $y=1$

  4. $y=-2$


Correct Option: A
Explanation:

We know equation of normal to the hyperbola $\cfrac{x^2}{a^2}-\cfrac{y^2}{b^2}=1$ is given by, $\cfrac{a^2x}{x _1}+\cfrac{b^2y}{y _1}=a^2-b^2$

Thus, required normal is, $5x+\cfrac{16y}{0}=9$
Clearly denominator of second term of L.H.S is $0$ so the equation of line is, $y=0$ 
Hence, option 'A' is correct.  

Find the equation of normal to the hyperbola $\displaystyle \frac{x^2}{16}\, -\displaystyle 
\frac{y^2}{9}=1$ at the point $\left ( 6, \displaystyle \frac{3}{2}\sqrt{5}\,\right )$

  1. $8\, \sqrt{5}x\, +\, 18y\, =\, 75\, \sqrt{5}$

  2. $4\, \sqrt{5}x\, +\, 9y\, =\, 25\, \sqrt{5}$

  3. $4\, \sqrt{5}x\, +\, 9y\, =\, 75\, \sqrt{5}$

  4. $8\, \sqrt{5}x\, +\, 18y\, =\, 25\, \sqrt{5}$


Correct Option: A
Explanation:

Required normal is given by,
$\displaystyle \frac{a^2x}{x _1}+\frac{b^2y}{y _1}=a^2+b^2$
$\Rightarrow \displaystyle \frac{16x}{6}+\frac{9y}{(3\sqrt{5}/2)}=25$
$\Rightarrow 8\, \sqrt{5}x\, +\, 18y\, =\, 75\, \sqrt{5}$

If e and e' be the eccentricities of a hyperbola and its conjugate, then $\displaystyle \dfrac{1}{e^2} + \dfrac{1}{e'^2} $ is equal to

  1. 0

  2. 1

  3. 2

  4. None of these


Correct Option: B
Explanation:

Suppose $\displaystyle \dfrac{x^2}{a^2} - \dfrac{y^2}{b^2} = 1$ be a hyperbola and let $\displaystyle \dfrac{x^2}{a^2} - \frac{y^2}{b^2} = - 1$ be its conjugate.
Then their eccentricities are given by $e^2 = \displaystyle \dfrac{a^2 + b^2}{a^2}$ and $\displaystyle e'^2 = \frac{a^2 + b^2}{b^2}$ respectively.
$\therefore \displaystyle \dfrac{1}{e^2} + \dfrac{1}{e'^2} = \dfrac{a^2}{a^2 + b^2} + \dfrac{b^2}{a^2 + b^2} = 1$

The normal to a curve at $P(x, y)$ meets the x-axis at $G$. If the distance of $G$ from the origin is twice the abscissa of $P$, then the curve is :

  1. an ellipse

  2. a parabola

  3. a circle

  4. a hyperbola


Correct Option: A,D
Explanation:

Equation of normal $\displaystyle Y-y=-\frac { dy }{ dx } \left( X-x \right) $
$\displaystyle \Rightarrow G=\left( x+y\frac { dy }{ dx } ,0 \right) $
According to question
$\displaystyle \left| x+y\frac { dy }{ dx }  \right| =\left| 2x \right| \Rightarrow y\frac { dy }{ dx } =x$ or $\displaystyle y\frac { dy }{ dx } =-3x$
$\Rightarrow ydy=xdx$ or $ydy=-3xdx$
$\displaystyle \Rightarrow \frac { { y }^{ 2 } }{ 2 } =\frac { { x }^{ 2 } }{ 2 } +c$ or $\displaystyle \frac { { y }^{ 2 } }{ 2 } =-\frac { 3{ x }^{ 2 } }{ 2 } +c$
$\Rightarrow { x }^{ 2 }-{ y }^{ 2 }=-2c$ or $3{ x }^{ 2 }+{ y }^{ 2 }=2c$

lf the line $ax+by+c=0$ is a normal to the curve $xy=1$, then  :

  1. $a>0,b>0$

  2. $a>0,b<0$

  3. $a<0,b>0$

  4. $a<0,b<0$


Correct Option: B,C
Explanation:

$xt^{3}-yt-t^{4}+120$
$eq^{n}$ of the normal at $t$ will same as the line $ax+by+c=0$
$\therefore \dfrac{t^{3}}{a}=\dfrac{-t}{b}=\dfrac{-1-t^{4}}{c}$
$t^{2}=\dfrac{-a}{b}$
$\therefore ab<0$

The equation of the normal at the positive end of the latusrectum of the hyperbola $x^2-3y^2=144$ is

  1. $\sqrt{3}x+2y=32$

  2. $\sqrt{3}x-3y=48$

  3. $3x+\sqrt{3}y=48$

  4. $3x-\sqrt{3}y=48$


Correct Option: A
Explanation:
The given hyperbola has the equation $\dfrac{x^2}{12^2}-\dfrac{y^2}{(4\sqrt{3})^2}=1$
Eccentricity of hyperbola = $e = \dfrac{\sqrt{a^2+b^2}}{a}=\dfrac{2}{\sqrt{3}}$
Now, equation of positive latus rectum is $x=ae=8\sqrt{3}$
The end-points of latus rectum are calculated as
$\dfrac{(8\sqrt{3})^2}{12^2}-\dfrac{y^2}{(4\sqrt{3})^2}=1$
$\therefore \dfrac{16}{12}-\dfrac{y^2}{48}=1$
$\therefore y^2=\dfrac{48}{3}$
$\therefore y=\pm 4$
Hence, the positive end is $(8\sqrt{3},4)$.
Now, equation of normal at any point $(x _1,y _1)$ is $\dfrac{a^2x}{x _1}+\dfrac{b^2y}{y _1}=a^2e^2$
$\therefore \dfrac{144x}{8\sqrt{3}}+\dfrac{48y}{4}=48\times 4$
$\therefore 6\sqrt{3}x+12y=48\times 4$
$\therefore \sqrt{3}x+2y=32$
This is the required answer.

Which one of the following points does not lie on the normal to the hyperbola, $\cfrac { { x }^{ 2 } }{ 16 } -\cfrac { { y }^{ 2 } }{ 9 } =1$ drawn at the point $\left( 8,3\sqrt { 3 }  \right) $?

  1. $\left( 13,-\cfrac { 1 }{ \sqrt { 3 } } \right) $

  2. $\left( 12,\cfrac { 1 }{ \sqrt { 3 } } \right) $

  3. $\left( 11,\sqrt { 3 } \right) $

  4. $\left( 10,\sqrt { 3 } \right) $


Correct Option: D
Explanation:

$\dfrac{x^2}{16} - \dfrac{y^2}{9} = 1$   $\Rightarrow \dfrac{2x}{16} - \dfrac{2y}{9} \dfrac{dy}{dx} = 0$

At $(8, 3\sqrt{3})$
$\dfrac{2\times 8}{16} - \dfrac{2\times 3\sqrt{3}}{9}\dfrac{dy}{dx}=0 \Rightarrow \dfrac{3}{2\sqrt{3}}=\dfrac{dy}{dx}$

Therefore slope of normal $=-\dfrac{1}{\frac{dy}{dx}}$
Equation of normal at $(8, 3\sqrt{3})$, 
$y-3\sqrt{3} = -\dfrac{2}{\sqrt{3}}(x-8)$
Clearly, option (D) does not lies on it.

Let $A\left( A\sec { \theta  } ,3\tan { \theta  }  \right) $ and $B\left( A\sec { \phi  } ,3\tan { \phi  }  \right) $ where $\theta +\phi =\cfrac { \pi  }{ 2 } $, be two points on the hyperbola $\cfrac { { x }^{ 2 } }{ 4 } -\cfrac { { y }^{ 2 } }{ 9 } =1$. If $\left( \alpha ,\beta  \right) $ is the point of intersection of normals to the hyperbola at $A$ and $B$, then $\beta=$

  1. $\cfrac { -13 }{ 3 } $

  2. $\cfrac { 13 }{ 3 } $

  3. $\cfrac { 3 }{ 13 } $

  4. $\cfrac { -3 }{ 13 } $


Correct Option: A
Explanation:

equation of hyperbola at point $A(2\sec{\theta} , 3\tan{\theta})$ is

$y+\dfrac{2}{3}\sin{\theta}x = \dfrac{13}{3}\tan{\theta}$   -------  $(i)$

and at point $B(2sec{\phi} , 3\tan{\phi})$ is
$y+\dfrac{2}{3}\sin{\phi}x = \dfrac{13}{3}\tan{\phi}$
now 

putting $\phi = \dfrac{\pi}{2} - \theta$


$y+\dfrac{2}{3}\cos{\theta}x = \dfrac{13}{3}\cot{\theta}$   -----  $(ii)$

now multiplying eq.(i) with  $\cos{\theta}$   and eq (ii) with  $\sin{\theta}$

then subtract both equation we find value of $\beta = -\dfrac{13}{3}$

If the sum of the slopes of the normal from a point P to the hyperbola $xy = {c^2}$is equal to $\lambda (\lambda  \in {R^ + })$,then the locus of point P is 

  1. ${x^2} = \lambda {c^2}$

  2. ${y^2} = \lambda {c^2}$

  3. ${xy} = \lambda {c^2}$

  4. ${y^2} = {c^2}$


Correct Option: A
Explanation:
Equation of rectangular hyperbola is $xy={c}^{2}$

Its rectangular coordinates are $\left(ct,\dfrac{c}{t}\right)$

Equation of normal is $c{t}^{4}-x{t}^{3}+ty-c=0$

Slope$=\dfrac{-coefficient\,of\,x}{coefficient\,of\,y}=\dfrac{{t}^{3}}{t}={t}^{2}$

The normal passes through the point $P\left(h,k\right)$

$\Rightarrow\,c{t}^{4}-h{t}^{3}+tk-c=0$

$\therefore\,$ there exists $4$ roots ${t} _{1},{t} _{2},{t} _{3}$ and ${t} _{4}$

Sum of the roots$={t} _{1}+{t} _{2}+{t} _{3}+{t} _{4}=\dfrac{-coefficient\,of\,{t}^{3}}{coefficient\,of\,{t}^{4}}=\dfrac{-\left(-h\right)}{c}=\dfrac{h}{c}$

Sum of the roots taken two at a time$=\sum{{t} _{i}{t} _{j}}={t} _{1}{t} _{2}+{t} _{2}{t} _{3}+{t} _{3}{t} _{4}+{t} _{4}{t} _{1}+{t} _{2}{t} _{4}+{t} _{1}{t} _{3}=\dfrac{-coefficient\,of\,{t}^{2}}{coefficient\,of\,{t}^{4}}=\dfrac{0}{c}=0$

Now,$\sum{{{t} _{i}}^{2}}=\sum{{\left({t} _{i}\right)}^{2}}$ using ${a}^{2}+{b}^{2}={\left(a+b\right)}^{2}$ for $ab=0$

Sum of squares of slopes of normal from $P$ is
${{t} _{1}}^{2}+{{t} _{2}}^{2}+{{t} _{3}}^{2}+{{t} _{4}}^{2}={\left({t} _{1}+{t} _{2}+{t} _{3}+{t} _{4}\right)}^{2}$

$\Rightarrow\,\lambda={\left(\dfrac{h}{c}\right)}^{2}$

$\Rightarrow\,{h}^{2}=\lambda{c}^{2}$

Replace $h\rightarrow\,x$ we get

${x}^{2}=\lambda{c}^{2}$

$\therefore\,{x}^{2}=\lambda{c}^{2}$ is the required locus at $P$

Let $P\left( a\sec { \theta  } ,b\tan { \theta  }  \right) $ and $Q\left( a\sec { \phi  } ,b\tan { \phi  }  \right) $, where $\theta +\phi =\dfrac {\pi}{2} $, be the two points on the hyperbola $\cfrac { { x }^{ 2 } }{ { a }^{ 2 } } -\cfrac { { y }^{ 2 } }{ { b }^{ 2 } } =1$. If $(h,k)$ is the point of intersection of the normals of $P$ and $Q$, then $k$ is equal to

  1. $\dfrac { { a }^{ 2 }+{ b }^{ 2 } }{ a } $

  2. $-\left[\dfrac { { a }^{ 2 }+{ b }^{ 2 } }{ a }\right] $

  3. $\dfrac { { a }^{ 2 }+{ b }^{ 2 } }{ { b } } $

  4. $-\left[\dfrac { { a }^{ 2 }+{ b }^{ 2 } }{ b } \right]$


Correct Option: D
Explanation:

The equations of the normal at P is $ax+bycosec\theta =\left ( a^{2}+b^{2} \right )\sec \theta $          (i)

and the equation of the normal at $Q\left ( a\sec \phi , b\sec \phi  \right )$ is
$ax+by cosec\phi =\left ( a^{2}+b^{2} \right )\sec \phi $          (ii)
Subtracting (ii) from (i) we get

   $\displaystyle y=\frac{a^{2}+b^{2}}{b}.\frac{\sec \theta -\sec \phi }{cosec \theta -cosec \phi }$

So $\displaystyle k=y=\frac{a^{2}+b^{2}}{b}.\frac{\sec \theta -\sec

\left ( \pi /2-\theta  \right )}{cosec \theta -cosec \left ( \pi

/2-\theta  \right )}$          $\left [ \because \theta +\phi =\pi /2

\right ]$
    
$\displaystyle =\frac{a^{2}+b^{2}}{b}.\frac{\sec

\theta -cosec \theta }{cosec \theta -\sec \theta }=-\left [

\frac{a^{2}+b^{2}}{b} \right ]$
Hence, option 'D' is correct.

If a normal of slope $m$ to the parabola ${ y }^{ 2 }=4ax$ touches the hyperbola ${ x }^{ 2 }-{ y }^{ 2 }={ a^2 }$, then

  1. ${ m }^{ 6 }-{ 4m }^{ 4 }-{ 3m }^{ 2 }+1=0$

  2. ${ m }^{ 6 }-{ 4m }^{ 4 }+{ 3m }^{ 2 }-1=0$

  3. ${ m }^{ 6 }+{ 4m }^{ 4 }-{ 3m }^{ 2 }+1=0$

  4. ${ m }^{ 6 }+{ 4m }^{ 4 }+{ 3m }^{ 2 }+1=0$


Correct Option: D
Explanation:

Equation of normal with slope $'m'$ to the parabola $y^2=4ax$ is given by,
$y=mx-2am-am^3$
Also this line touches the hyperbola $x^2-y^2=a^2$
thus using condition of tangency to the hyperbola, $c^2=a^2m^2-b^2$
$(-2am-am^3)^2=a^2(m^2-1)$
$\Rightarrow 4m^2+m^6+4m^4=m^2-1$
$\Rightarrow m^6+4m^4+3m^2+1=0$
Hence, option 'D' is correct.

If a normal of slope $m$ to the parabola $y^2 = 4ax$ touches the hyperbola $x^2 - y^2 = a^2$, then

  1. $m^6 - 4m^4 - 3m^2 + 1 =0$

  2. $m^6 - 4m^4 + 3m^2 - 1 = 0$

  3. $m^6 + 4m^4 - 3m^2 + 1 = 0$

  4. $m^6 + 4m^4 + 3m^2 + 1 = 0$


Correct Option: D
Explanation:

Equation of normal with slope $'m'$ to the parabola $y^2=4ax$ is given by,
$y = mx-2am-am^3$ (i)

Now given (i) is tangent to the hyperbola $x^2-y^2=a^2$

Thus using condition of tangency, $c^2= a^2m^2-a^2$

$\Rightarrow (2am+am^3)^2=a^2(m^2-1)$

$\Rightarrow (2m+m^3)^2=m^2-1\Rightarrow m^6+4m^4+3m^2+1=0$

Let P $(asec \theta,\, btan \theta)$ and Q $(asec \phi,\, btan \phi)$, where $\theta\, +\, \phi\, =\, \displaystyle \frac{\pi}{2}$, be two points on the hyperbola $\displaystyle \frac{x^2}{a^2}\, -\, \frac{y^2}{b^2}\, =\, 1$. If (h, k) is the point of intersection of the normals at P & Q, then k is equal to

  1. $\displaystyle \frac{a^2\, +\, b^2}{a}$

  2. $\displaystyle - \left (\frac{a^2\, +\, b^2}{a}\right )$

  3. $\displaystyle \frac{a^2\, +\, b^2}{b}$

  4. $\displaystyle - \left (\frac{a^2\, +\, b^2}{b}\right )$


Correct Option: D
Explanation:

Normal at $\theta,\, \phi$ are
$\displaystyle \left {

\begin{matrix} ax\, cos\, \theta & + & by\, cot\, \theta\, =\,

a^2\, +\, b^2 \ ax\, cos\, \phi & + & by\, cot\, \phi\, =\,

a^2\, +\, b^2 \end{matrix}\right.$
where $\displaystyle \phi\, =\, \frac{\pi}{2}\, -\, \theta$ and these passes through (h, k).

$\therefore\, ah\, cos \theta\, +\, bk\, cot \theta\, =\, a^2\, +\, b^2$ .....(i)
$ah\, sin \theta\, +\, bk\, tan \theta\, =\, a^2\, +\, b^2$ .....(ii)
Multiply (i) by $sin \theta$ & (ii) by $cos \theta$ & subtract them, 
we get
$\Rightarrow\, (bk\, +\, a^2\, +\, b^2)\, (sin \theta\, -\, cos \theta)\, =\, 0$
$k =-(\cfrac{a^2 + b^2}{b})$
Hence, option 'D' is correct.

From any point R two normals which are right angled to one another are drawn to the hyperbola $\displaystyle \frac{x^{2}}{a^{2}}-\frac{y^{2}}{b^{2}}=1,\left ( a>b \right )$ If the feet of the normals are P and Q then the locus of the circumcentre of the triangle PQR is

  1. $\displaystyle \frac{x^{2}+y^{2}}{a^{2}-b^{2}}=\left ( \frac{x^{2}}{a^{2}}+\frac{y^{2}}{b^{2}} \right )^{2}$

  2. $\displaystyle \frac{x^{2}-y^{2}}{a^{2}-b^{2}}=\left ( \frac{x^{2}}{a^{2}}-\frac{y^{2}}{b^{2}} \right )^{2}$

  3. $\displaystyle \frac{x^{2}+y^{2}}{a^{2}-b^{2}}=\left ( \frac{x^{2}}{a^{2}}-\frac{y^{2}}{b^{2}} \right )^{2}$

  4. $\displaystyle \frac{x^{2}+y^{2}}{a^{2}+b^{2}}=\left ( \frac{x^{2}}{a^{2}}-\frac{y^{2}}{b^{2}} \right )^{2}$


Correct Option: C
Explanation:

Clearly tangent at P and Q intersect at right-angles at S (say)
$ \displaystyle \Rightarrow $ PSQR is cyclic


$ \displaystyle \Rightarrow $ S lies on director circle of hyperbola

$ \displaystyle \Rightarrow S=\sqrt{a^{2}-b^{2}}\cos \theta , \sqrt{a^{2}-b^{2}}\sin \theta  $

$ \displaystyle \Rightarrow   $ Chord with middle point (h,k) i.e. circumcentre will be same as equation of chord of contact w.r.$ \displaystyle \Rightarrow \perp  $ s

$ \displaystyle \Rightarrow \frac{xh}{a^{2}}-\frac{yk}{b^{2}}=\frac{h^{2}}{a^{2}}-\frac{k^{2}}{b^{2}}$ and $\frac{x\sqrt{a^{2}-b^{2}\cos \theta }}{a^{2}}-\frac{y\sqrt{a^{2}-b^{2}}\cos\theta }{b^{2}}=1 $ are identical comparing and solving we get locus as $ \displaystyle \frac{x^{2}+y^{2}}{a^{2}-b^{2}}=\left ( \frac{x^{2}}{a^{2}}-\frac{y^{2}}{b^{2}} \right )^{2} $

- Hide questions